What is the critical value z* for constructing an 80% confidence interval?

Answers

Answer 1

Answer:khan is 1.282 or c

Step-by-step explanation:


Related Questions

Erick no es mas alto que andres, oscar es mas bajo que carlos y este no es mas alto que erick. se puede deducir que:
a. Andrés es más alto que Erick
b. Carlos es más alto que Andrés
c. Oscar es más alto que Carlos
d. Oscar es el más bajo de todos
e. Ninguna de las anteriores

Answers

Answer:

es la b) Oscar es más bajos que todos

Step-by-step explanation:

espero y te ayude

HELP ME PLEASE I need it

Answers

The answer should be
f=1/5 x^2 + -2/5x

Brent works part-time at a clothing store. He earns an hourly wage of $15. If he needs to earn more than $45 in a day and works for x hours a day, which inequality represents this situation? A. 15x > 45 B. 15x < 60 C. x > 60 D. 15x < 45 E. x < 60

Answers

Answer:

A) 15x > 45

Step-by-step explanation:

Let X represent an hour. His wage will be 15 times that, so you have:

15x

He needs to earn more than 45 a day, so the inequality needs to be >45

Only answer that satisfies these conditions is A:

A) 15x > 45

Answer:     A.

Step-by-step explanation:

AB intersects CD at point E. What is the value of X in degrees?​

Answers

Answer:

x = 132°

Step-by-step explanation:

∠AED and ∠CEB are both vertical angles. Vertical angles will always be equal to each other, so ∠AED = ∠CEB.

Since they're equal, if ∠AED = 132°, then ∠CEB must also equal 132°.

(∠CEB is the x value you're asking about by the way).

If you have any questions, or anything was confusing, comment them and I'll help you out.

A tennis club has 560 members.
(a) The ratio
men : women: children
5:6:3
(i) Show that the club has 240 women members.

(ii) How many members are children?

Answers

Answer:

see explanation

Step-by-step explanation:

sum the parts of the ratio, 5 + 6 + 3 = 14 parts

Divide number of members by 14 to find the value of one part of the ratio.

560 ÷ 14 = 40 ← value of 1 part of the ratio

(i)

number of women = 6 × 40 = 240

(ii)

number of children = 3 × 40 = 120

Area please I’ll mark brainliest

Answers

Answer:

112

Step-by-step explanation:

A = (B1 + B2)(8)/2

A = 28(8)/2

A = 112

112 ur welcome <3

..............................................................................................................................................

Can someone help me, please!!

Answers

Id say its option 3.

I have tree pizzas. The radius of one pizza is 4 inches. The radius of the second pizza is 8 inches. The radius of the last pizza is 12 inches. If the first pizza can feed 2 people, how many people can the other pizzas feed?

Answers

10? Yea I think 10 is the answer

I really need help with this it’s on Aleks

Answers

Answer:

y = 2

Step-by-step explanation:

Given

[tex]\sqrt[3]{3y+2}[/tex] + 4 = 6 ( subtract 4 from both sides )

[tex]\sqrt[3]{3y+2}[/tex] = 2 ( cube both sides )

3y + 2 = 2³ = 8 ( subtract 2 from both sides )

3y = 6 ( divide both sides by 3 )

y = 2

Y would equal 2. You start off by subtracting 4 by both sides then you simply

Simplify (2.37x10^8)3

Answers

Answer:

( 2 . 3 7 1 0 8 ) 3

Step-by-step explanation:

The answer is 3. I don't feel like explaining it bc it's 3 in the morning, but you're welcome I guess.

What is the distance between the origin and the point (2,-5) on a graph? The options are: square root of 7. Square root of 29. 7. 29.

Answers

Answer:

Square root of 29.

Step-by-step explanation:

Distance = sqrt(2^2 + (-5)^2)

= sqrt 29.

David got an $80 gift card to walmart. He uses the gift card to buy a dog toy for $12.99. He also wants to use the gift card to buy some candy bars. Each candy bar cost $0.99 how many candy bars can he buy

Answers

Answer:

67.68

68 rounded

Step-by-step explanation:

80-12.99=67.01

67.01 divided by 0.99=67.68 repeating

Find the surface area of the figure below

Answers

Answer:

Step-by-step explanation:

radius r=16/2=8 in

l=17 in

curved surface=πrl=π×8×17=136 π in²

area of circular base=πr²=π×8²=64 πin²

total surface area=136 π+64 π=200 π in²

ill give brainlist for you will receive the 30 if you get them all 3 right if you get one right ill give you thanks if you do 2 ill give thanks and all 5 hearts and if you get all 3 right ill do brainlist and the thanks as well as the 5 hearts

Answers

Answer:

The answers from left side on

X=60

X=15

X=21

Step-by-step explanation:

Good luck ^_^

I don’t know if this is correct !!!!!!!!!! Please answer correctly !!!!!!!!!!! Will mark Brianliest !!!!!!!!!!!!!!!!!!!

Answers

Answer: They are related by the AAS postulate and they can be proven congruent.

Step-by-step explanation: You can see that one of the lines on each of the triangle has that little tick which means those lines are congruent. You can also see that for two of the angles one of them has one arc and the other has two arcs for BOTH triangles which makes those two angles congruent. Two angles and one side congruent makes the triangles congruent by the AAS postulate.

I hope I helped! :)

The equation for the perimeter around Claire's garden is a:
O Linear Function
O Exponential Function
O Quadratic Function
O Absolute Value Function

Answers

Answer:

Linear function

Step-by-step explanation:

Took da test ;)

A rectangular box has a volume of 15 cubic feet. The length, width, and height of the box are each doubled.
What is the new volume of the rectangular box?
A. 30 cubic feet
B. 60 cubic feet
C. 90 cubic feet
O D. 120 cubic feet

Answers

Answer:

it should be D/120

Step-by-step explanation:

An example of the box is:

5 the width

3 the length

1 the height.

That would be 5 x 3 x 1.

To double them, you must multiply each number by 2:

5 x 2 = 10

3 x 2 = 6

1 x 2 = 2.

Now the equation is 10 x 6 x 2.

First multiply 10 x 6 (60).

Then 60 x 2 (120).

120 is the answer

In a small town of 5,832 people, the mayor wants to determine the proportion of voters who would support an increase to the food tax. An assistant to the mayor decides to survey 1,000 randomly chosen people to construct a 90% confidence interval for the true proportion of people who would support the increase in food tax. Of the sample, 363 people say they would support the increase. Are the conditions for inference met?

Yes, the conditions for inference are met.
No, the 10% condition is not met.
No, the randomness condition is not met.
No, the Large Counts Condition is not met.

Answers

Answer:

No, the 10% condition is not met is the answer.

1,000/5,832 is 0.1714, or ~17%, which is more than 10% of the population.

No, the 10% condition is not met.

Ratio:

'Ratio is a term that is used to compare two or more numbers. It is used to indicate how big or small a quantity is when compared to another.'

Proportion:

'A proportion is an equation in which two ratios are set equal to each other.'

According to the given problem,

Total number of people in the town = 5832

Number of people to be surveyed = 1000

Ratio of number of people chosen to the total population = [tex]\frac{1000}{5832}[/tex]

                                                                                                 = 0.17

Therefore percentage of people to be surveyed = 0.17 × 100

                                                                         = 17%, which is more than 10%

Hence, we can conclude that the 10% condition is not met.

Learn more about Ratio and Proportion here:

https://brainly.in/question/30210407

#SPJ2

Please help!: Mrs. Carter baked a cake that was in the shape of a rectangular prism. The cake was 24 inches long, 15 inches wide and 3 inches high. She spread frosting on all four sides and the top. How many square inches of frosting did she use?

Answers

rectangle base: 24 x 15 = 360

triangle: 15 x 3 x 1/2= 45 x 1/2 = 22.5 x 4 (each side) 90 inches

total: 360 + 90 = 450 inches :)

?!??!!!!!someone help me

Answers

Answer:

its b

Step-by-step explanation:

Well the easy way to find this is to just solve the first one. the equation is

x-y>5 so you plug in the coordinate and you end up with 7-2>5 which is false. So any answer with option one is out and b is the only one that fits that description.

Please help best gets brainlist and 5/5 and show work please

Answers

The answer is 12ft

Volume = (base area)(height)
900 = (75)(x)

So to figure out x you’ll need to divide 900 by 75 to get 12

Hope this helps!

Please help due soon and explain why it’s that choice. Thanks! Giving brainiest btw.

Answers

Rational numbers can’t be integers cause integer numbers can’t be written with fraction or decimal it just a whole number like 3
So I guess C

PLEASE HELPP!!I need to pass this please!!im confused​

Answers

Answer:

b

Step-by-step explanation:

i believe the answer is b

Simplify.

[tex]\sqrt{289}[/tex] fast plss

Answers

Answer:

17 is the answer

Step-by-step explanation:

Since 17 × 17 is 289, the square root of 289 is 17.

so therefor, the answer is 17.

3.8n+1=3.6m+1.62 (show the work)

Answers

Answer:

M=1.05n-0.172

Step-by-step explanation:

Determine which ordered pair is a solution to the inequality: 3x+y>6

(1,1) (2,1) (1,2) (0,0)

Answers

Answer:

order pair (2,1) is the solution to the inequality.

8 km
6 km
What is the length of the hypotenuse?
kilometers
C

Answers

Answer:

10

Step-by-step explanation:

We use pythagorean theorem:

a^2 + b^2 = c^2

8^2 + 6^2 = c^2

[tex]\sqrt{36 + 64} = \sqrt{100} = 10[/tex]

please help!! due soon, i’ll give brainliest

Answers

Answer:

4

Step-by-step explanation:

an aquarium is in the shape of a cuboid

the depth can be determined from the volume of a cuboid

volume = length x depth x height

11.96 = 2.5 x 1.1 x depth

depth = 11.96 / (2.5 x 1.1 )

depth = 4.3 feet = 4 (to the nearest whole number)

To round off to the nearest whole number, look at the first number after the decimal, if it is less than 5, add zero to the units term, If it is equal or greater than 5, add 1 to the units term.

help with segment relationships in circles...picture attached.

Answers

The answer: (D)

Explorer acc nss as time yyy has a new one in the middle school in the school and is the junior high junior in junior high junior college

can you simplify -3(x+7)

Answers

Answer:-3x-21

Step-by-step explanation: use the distributive property

Answer:

=−3x−21

Step-by-step explanation:

−3(x+7)

=(−3)(x+7)

=(−3)(x)+(−3)(7)

=3x-21

Other Questions
PLEASE HELP ME!!!!IT IS FOR BIOLOGY!!! help me please?The function (x) = 2x + 1 indicates: a downward shift of 1 unit. a shift of 1 unit to the left an upward shift of 1 unit. None of these choices are correct. Help!!! Complete the following sentences with the appropriate indirect object and direct object pronouns. How many sides does a 27-gon have? Which expression is equivalent to m2 13m 30? john takes a small business loanfor $18,000. the term forrepayment is 60 months andthe annual interest rate is 5.25%.using the monthly paymentformula, what is the estimatedmonthly payment? Summarize the necklace short story by guy de Maupassant Which adjective correctly completes this sentence? Miguel es ----.A. reservada B. deportista C. simptica D. artistica I need help with history please Identify parts of the E-Cig that constitute voltage, current, and resistance. Discuss the role each plays in the E-Cig and typical values for each including units.Discuss the electrical dangers of an E-Cig. Give specific examples. There are many electrical safety rules. Pick one, and discuss its application on a small system, such as the E-Cig. 5 word summary of Britain enacts Reforms Decide if the following sentence is grammatically CORRECT or INCORRECT.Je parle le franais.O CorrectO Incorrect Last year, Maria planted a tree that is 5 11/12 feet tall. This year, the tree is 7 2/3 feet tall. How many feet did the tree grow? PLSS HELP ME!! I WILL GIVE YOU BRAINLYEST!!Howard Hughes invited a drill bit that allowed for drilling deeper i rock. What industry would have benefited from this? Is this correct? Can someone help me pls HELPPPPP IM GIVING 50 POINTS FOR THIS ! A jar contains 40 balls numbered through 40. Three balls are removed without being returned to the jar, and the numbers are written down in order. How many sequences of numbers are possible? 0369Y57911Find the slope of the line represented by the table of values.A)2.B)3C)2/3D)3/2 1. Choose the justification for the step below.7(3+x)=1421+77= 14a) additive property of equalityb) multiplication/division property of equalityc) distributiond) none of these Aliens definition ?